Đến nội dung

Hình ảnh

[TOPIC] ÔN THI BẤT ĐẲNG THỨC $\boxed{\text{THPT CHUYÊN}}$ LỚP $10$ năm $2018-2019$

bất đẳng thức holder cosi bunhiacopxki

  • Chủ đề bị khóa Chủ đề bị khóa
Chủ đề này có 318 trả lời

#161
DinhXuanHung CQB

DinhXuanHung CQB

    Trung sĩ

  • Thành viên
  • 118 Bài viết

Bài 69: Cho $a,b,c>0$ và $abc=1$. Chứng minh rằng $\sqrt{a^{2}-a+1}+\sqrt{b^{2}-b+1}+\sqrt{c^{2}-c+1}\geq a+b+c$

Đây là đề thi Olympic $30/4$ khối $10$ năm nay. Bạn tìm đọc lời giải nhé


Little Homie


#162
tr2512

tr2512

    Thượng sĩ

  • Thành viên
  • 272 Bài viết

Bài 73

Cho 3 số a, b, c thực dương. Chứng minh:

$ a^3+b^3+c^3+3abc \ge ab\sqrt{2(a^2+b^2)} + bc\sqrt{2(b^2+c^2)} + ac\sqrt{2(a^2+c^2)} $

 

 

@DinhXuanHung CQB: e có một lời giải khác cho bài 30/04 nhưng phải dùng tới đạo hàm nên không đăng được :D chán


Bài viết đã được chỉnh sửa nội dung bởi tr2512: 22-04-2018 - 19:37


#163
MoMo123

MoMo123

    Sĩ quan

  • Điều hành viên THCS
  • 334 Bài viết

 

Bài 53. Cho $a\,,\,b\,,\,c$ là 3 cạnh của 1 tam giác. Chứng minh rằng: $\frac{a^{3}}{b}\,+\, \frac{b^{3}}{c}\,+ \,\frac{c^{3}}{a}\,\geqq \,\frac{3\,(a^{2}\,+\, b^{2}\,+\, c^{2})}{a\,+\, b\,+\, c}$

 

$(a+ b+ c)\,(\frac{a^{3}}{b}+ \frac{b^{3}}{c}+ \frac{c^{3}}{a})$

 

$=\,(a^{3}+ b^{3}+ c^{3})\,+ \,(\frac{a^{4}}{b}+ \frac{b^{4}}{c}+ \frac{c^{4}}{a})\,+\, (\frac{ab^{3}}{c}+ \frac{bc^{3}}{a}+ \frac{ca^{3}}{b})$

 

$\geqq \,(a^{3}+ b^{3}+ c^{3})\,+ \,(\frac{a^{4}}{b}+ \frac{b^{4}}{c}+ \frac{c^{4}}{a})\,+\, (a^{2}b+ b^{2}c+ c^{2}a)$

 

$= \,(a^{3}+ b^{3}+ c^{3})\,+ \,(\frac{a^{4}}{b}+ a^{2}b)\,+\, (\frac{b^{4}}{c}+ b^{2}c)\,+\, (\frac{c^{4}}{a}+ c^{2}a)$

 

$\geqq \,3\,(a^{3}+ b^{3}+ c^{3})$

 

Chà, cái này sao đáp giải lại khác với đề vậy ạ, mong anh xem lại



#164
tr2512

tr2512

    Thượng sĩ

  • Thành viên
  • 272 Bài viết

Bài 74: 

Cho a, b, c là 3 số thực không âm. Chứng minh:

$\frac{a^4}{a^2+ab+b^2}+\frac{b^4}{b^2+bc+c^2}+\frac{c^4}{c^2+ac+c^2} \ge \frac{a^3+b^3+c^3}{a+b+c} $

 

@Momo123: đề ban đầu chắc bị DOTOANANG gõ nhầm, thử (a,b,c) =(0.11; 0.5; 0.4) vào đề lúc đầu thấy VT<VP mà :D 



#165
phamhuy1801

phamhuy1801

    Trung sĩ

  • Thành viên
  • 181 Bài viết

Bài 74: 

Cho a, b, c là 3 số thực không âm. Chứng minh:

$\frac{a^4}{a^2+ab+b^2}+\frac{b^4}{b^2+bc+c^2}+\frac{c^4}{c^2+ac+c^2} \ge \frac{a^3+b^3+c^3}{a+b+c} $

 

$\frac{a^4}{a^2+ab+b^2}+\frac{b^4}{b^2+bc+c^2}+\frac{c^4}{c^2+ac+c^2} \ge \frac{a^3+b^3+c^3}{a+b+c} $
$\Leftrightarrow \frac{a^4}{a^2+ab+b^2}-a(a-b)+\frac{b^4}{b^2+bc+c^2}-b(b-c)+\frac{c^4}{c^2+ac+c^2}-c(c-a) \ge \frac{a^3+b^3+c^3-(a+b+c)(a^2+b^2+c^2-ab-bc-ca)}{a+b+c} $
$\Leftrightarrow \frac{ab^3}{a^2+ab+b^2}+\frac{bc^3}{b^2+bc+c^2}+\frac{ca^3}{c^2+ac+c^2} \ge \frac{3abc}{a+b+c}$
 
Bất đẳng thức trên đúng vì:
 
$\frac{ab^3}{a^2+ab+b^2}+\frac{bc^3}{b^2+bc+c^2}+\frac{ca^3}{c^2+ac+c^2} = \frac{ab^3c}{c(a^2+ab+b^2)}+\frac{abc^3}{a(b^2+bc+c^2)}+\frac{bca^3}{b(c^2+ac+c^2)} \ge \frac{(\sum a\sqrt{abc})^2}{\sum a(b^2+bc+c^2)} = \frac{abc(a+b+c)^2}{(a+b+c)(ab+bc+ca)} \ge \frac{abc.3(ab+bc+ca)}{(a+b+c)(ab+bc+ca)} = \frac{3abc}{a+b+c} $

Bài viết đã được chỉnh sửa nội dung bởi phamhuy1801: 22-04-2018 - 20:29


#166
hoangkimca2k2

hoangkimca2k2

    Sĩ quan

  • Thành viên
  • 477 Bài viết

Đây là đề thi Olympic $30/4$ khối $10$ năm nay. Bạn tìm đọc lời giải nhé

 

Một cách khác không cần dùng đạo hàm.

 

Dùng BDT $Bunhiacopxki$

Ta có: $(a^{2}+a+1)\sqrt{a^{2}-a+1}=\sqrt{(a^{2}+a+1)(a^{4}+a^{2}+1)}=\sqrt{\left [(a+\frac{1}{2})^{2}+\frac{3}{4}  \right ]\left [ (a^{2}+\frac{1}{2})^{2}+\frac{3}{4} \right ]}$

$\geq a^{3}+\frac{1}{2}(a^{2}+a)+1$

$\Rightarrow \sqrt{a^{2}-a+1}-a\geq \frac{1}{2}(\frac{3}{a^{2}+a+1}-1)$

Cái sau là BDT $Vasile$ quen thuộc.


  N.D.P 

#167
MoMo123

MoMo123

    Sĩ quan

  • Điều hành viên THCS
  • 334 Bài viết

Bài 67: Cho $a,b,c>0$. Chứng minh rằng $\sqrt{(a^{2}b+b^{2}c+c^{2}a)(ab^{2}+bc^{2}+ca^{2})}\geq abc+\sqrt[3]{(a^{3}+abc)(b^{3}+abc)(c^{3}+abc)}$

Chia cả 2 vế cho abc , ta có bất đẳng thức tương đương 

$\sqrt{(\frac{a}{c}+\frac{b}{a}+\frac{c}{b})(\frac{b}{c}+\frac{c}{a}+\frac{a}{b})} \geq 1+\sqrt[3]{(\frac{a^2}{bc}+1)(\frac{b^2}{ac}+1)(\frac{c^2}{ab}+1)}$

Đặt $(\frac{a}{b};\frac{b}{c};\frac{b}{c})=(x,y,z)$

$ -> xyz=1$

Bất đẳng thức $\Leftrightarrow$ $\sqrt{(xy+yz+zx)(x+y+z)} \geq 1+\sqrt[3]{(x+y)(y+z)(z+x)}$

$\Leftrightarrow \sqrt{(x+y)(y+z)(z+x)+1} \geq \sqrt[3]{(x+y)(y+z)(z+x)}$

Đặt $\sqrt[3]{(x+y)(y+z)(z+x)} = m $

Ta cần chứng minh $\sqrt{m^3+1} \geq m+1$

$ \Leftrightarrow m(m+1)(m-2) \geq 0 $

Bất đẳng thức cuối cùng đúng vì $m=\sqrt[3]{(x+y)(y+z)(z+x)} \geq \sqrt[3]{8xyz} =2$



#168
xuanhoan23112002

xuanhoan23112002

    Trung sĩ

  • Thành viên
  • 103 Bài viết

Bài 75: Cho a, b, c là các số thực dương. CMR:

$a^2+b^2+c^2+2abc+1\geq 2(ab+bc+ca)$



#169
tr2512

tr2512

    Thượng sĩ

  • Thành viên
  • 272 Bài viết

Bài 75: Cho a, b, c là các số thực dương. CMR:

$a^2+b^2+c^2+2abc+1\geq 2(ab+bc+ca)$

Một kết quả vô cùng quen thuộc

Theo nguyên lý Dirichlet, trong 3 số a, b, c luôn tồn tại 2 số nằm cùng bên với 1. Giả sử đó là a, b, ta có:\\
$ c(a-1)(b-1) \ge 0 $
$ \Leftrightarrow abc \ge c(a+b)-c $
Áp dụng vào bài toán
$ a^2+b^2+c^2+2abc+1 \ge a^2+b^2+c^2+2c(a+b)-2c+1 $
Vậy ta cần chứng minh:
$  a^2+b^2+c^2+2c(a+b)-2c+1 \ge 2ab+2bc+2ca $
$ \Leftrightarrow (a-b)^2+(c-1)^2 $ Luôn đúng 
Hoàn tất chứng minh.
P/s: bạn nên đưa ra nguồn gốc bài toán.


#170
tr2512

tr2512

    Thượng sĩ

  • Thành viên
  • 272 Bài viết

Bài 76: 

Cho 3 số thực a, b, c thỏa mãn $ abc \ge 0$. Chứng minh bất đẳng thức:
$ a^2+b^2+c^2+2abc+4 \ge 2(a+b+c)+ab+bc+ca $ (Lê Khánh Sỹ)

Bài viết đã được chỉnh sửa nội dung bởi tr2512: 23-04-2018 - 20:57


#171
xuanhoan23112002

xuanhoan23112002

    Trung sĩ

  • Thành viên
  • 103 Bài viết

Bài 77(APMO 2004): Cho a, b, c là các số thực dương. CMR: $(a^2+2)(b^2+2)(c^2+2)\geq 9(ab+bc+ca)$



#172
xuanhoan23112002

xuanhoan23112002

    Trung sĩ

  • Thành viên
  • 103 Bài viết

Bài 78(IMO 1984): Cho a, b, c là các số không âm thỏa mãn $a+b+c=1$.CMR:

$0\leq ab+bc+ca-2abc\leq \frac{7}{27}$



#173
Khoa Linh

Khoa Linh

    Thiếu úy

  • Thành viên
  • 601 Bài viết

Bài 77(APMO 2004): Cho a, b, c là các số thực dương. CMR: $(a^2+2)(b^2+2)(c^2+2)\geq 9(ab+bc+ca)$

Ta đi chứng minh kết quả mạnh hơn là $(a^2+2)(b^2+2)(c^2+2)\geq 3(a+b+c)^2$

Áp dụng Cauchy - Schwarz ta có:

$3(a+b+c)^2=3\left ( \frac{a+b}{\sqrt{2}}.\sqrt{2}+1.c \right )^2\leq 3\left ( \frac{(a+b)^2}{2}+1 \right )(c^2+2)$

Ta cần đi chứng minh: $(a^2+2)(b^2+2)\geq 3\left (\frac{(a+b)^2}{2}+1 \right )\Leftrightarrow \frac{(a-b)^2}{2}+(ab-1)^2\geq 0$ đúng 

Vậy hoàn tất chứng minh 


$\sqrt[LOVE]{MATH}$

"If I feel unhappy, I do mathematics to become happy. If I am happy, I

 

do mathematics to keep happy" - Alfréd nyi 


#174
xuanhoan23112002

xuanhoan23112002

    Trung sĩ

  • Thành viên
  • 103 Bài viết

Cách của bạn Linh đúng rồi mọi người thử tìm các cách khác chẳng hạn như dùng nguyên lí Đirichlet



#175
xuanhoan23112002

xuanhoan23112002

    Trung sĩ

  • Thành viên
  • 103 Bài viết

Bài 79: Cho a, b, c là các số thực dương thỏa mãn $a+b+c=3$. CMR:

$(a^2-a+1)(b^2-b+1)(c^2-c+1)\geq 1$



#176
thanhdatqv2003

thanhdatqv2003

    Trung sĩ

  • Thành viên
  • 159 Bài viết

bài mạnh hơn bài 77 đây

Bài 80 : Cho a,b,c là các số thực dương

C/m : $(a^2+3)(b^2+3)(c^2+3)\geq 4(a+b+c+1)^2$


Bài viết đã được chỉnh sửa nội dung bởi thanhdatqv2003: 24-04-2018 - 22:49

:ohmy: [Không tồn tại các nghiệm nguyên khác không x, y, và z thoả mãn xn + yn = zn trong đó n là một số nguyên lớn hơn 2.  (FERMAT)  :ohmy: 

 

 

 

 


#177
mduc123

mduc123

    Hạ sĩ

  • Thành viên
  • 55 Bài viết

Bài 78(IMO 1984): Cho a, b, c là các số không âm thỏa mãn $a+b+c=1$.CMR:

$0\leq ab+bc+ca-2abc\leq \frac{7}{27}$

Do a+b+c=1 và a,b,c không âm nên $ab+bc+ca-2abc=ab(1-c)+ca(1-b)+bc\geq 0$

Ta có: $ab+bc+ca-2abc=a(b+c)+bc(1-2a)\leq a(1-a)+\frac{(b+c)^{2}}{4}(1-2a)=a(1-a)+\frac{(a-1)^{2}}{4}(1-2a)$ 

Nên ta chỉ cần Cm $a(1-a)+\frac{(a-1)^{2}}{4}(1-2a)\leq \frac{7}{27}$ thật vậy $a(1-a)+\frac{(a-1)^{2}}{4}(1-2a)\leq \frac{7}{27}\Leftrightarrow (a+\frac{1}{6})(a-\frac{1}{3})^{2}\geq 0$ (đpcm)


Bài viết đã được chỉnh sửa nội dung bởi mduc123: 23-04-2018 - 12:46


#178
phamhuy1801

phamhuy1801

    Trung sĩ

  • Thành viên
  • 181 Bài viết

Bài $65$: Cho các số dương $x,y,z$. Chứng minh: 

                                               $(x+\frac{yz}{x})(y+\frac{zx}{y})(z+\frac{xy}{z}) \ge 4\sqrt[3]{(x^3+y^3)(y^3+z^3)(z^3+x^3)}$

 

Mình xin đưa ra lời giải cho bài toán của mình:

$(x+\frac{yz}{x})(y+\frac{zx}{y})(z+\frac{xy}{z}) \ge 4\sqrt[3]{(x^3+y^3)(y^3+z^3)(z^3+x^3)}$
Chia cả hai vế cho $xyz$ ta được:
$\Leftrightarrow (\frac{x}{y}+\frac{z}{x})(\frac{y}{z}+\frac{x}{y})(\frac{z}{x}+\frac{y}{z}) \ge 4\sqrt[3]{(\frac{x^3}{y^3}+1)(\frac{y^3}{z^3}+1)(\frac{z^3}{x^3}+1)} $
 
Đặt $(\frac{x}{y}; \frac{y}{z}; \frac{z}{x}) \rightarrow (a,b,c)$ thì $abc=1$ và ta sẽ chứng minh:
$\Leftrightarrow (a+b)(b+c)(c+a) \ge 4 \sqrt[3]{(a^3+1)(b^3+1)(c^3+1)}$
$\Leftrightarrow [(a+b)(b+c)(c+a)]^3 \ge 64(a^3+abc)(b^3+abc)(c^3+abc) $
$\Leftrightarrow \sqrt{[(a+b)(b+c)(c+a)]^3} \ge 8\sqrt{abc(a^2+bc)(b^2+ca)(c^2+ab)}$             $($  :D $)$
 
Xét bất đẳng thức: 
$(a+b)(b+c) = a^2+bc + b(c+a) \ge 2\sqrt{b(c+a)(a^2+bc)} $
Thiết lập các bất đẳng thức tương tự rồi nhân theo vế, ta thu được $[(a+b)(b+c)(c+a)]^2 \ge 8\sqrt{a(b+c)(a^2+bc).b(c+a)(b^2+ca).c(a+b)(c^2+ab)}$, đây chính là bất đẳng thức $($  :D $)$, bài toán được chứng minh.


#179
xuanhoan23112002

xuanhoan23112002

    Trung sĩ

  • Thành viên
  • 103 Bài viết

Bài 81(VMO 2015): Cho a, b, c là các số thực dương. CMR:

$3(a^2+b^2+c^2)\geq (a+b+c)(\sqrt{ab}+\sqrt{bc}+\sqrt{ca})+(a-b)^2+(b-c)^2+(c-a)^2\geq (a+b+c)^2$



#180
thanhdatqv2003

thanhdatqv2003

    Trung sĩ

  • Thành viên
  • 159 Bài viết

Cho a,b,c là 3 cạnh của tam giác

Bài 70: C/m: $P^2\geq 12\sqrt{3}S$        (với p là nửa chu vi, S là diện tích)

 

Bài 71: C/m :  $a^3+b^3+c^3 \geq $\frac{P^3}{9}$      (với P là chu vi)

 

Bài 72: C/m:  $a^3+b^3+c^3 \geq \frac{4\sqrt{3}}{3}SP$

Bài70   

Ta có $(p-a)(p-b)(p-c)\leq \frac{1}{27}\left [ (p-a)+(p-b)+(p-c) \right ]^3=\frac{1}{27}p^3$

Vậy $12\sqrt{3}S=12\sqrt{3}\sqrt{p(p-a)(p-b)(p-c)}\leq 12\sqrt{3}\sqrt{p\frac{1}{27}p^3}=4p^2=P^2$ (đpcm)

 

Bài 71

$P^3 =(a+b+c)^3=a^3+b^3+c^3+3ab(a+b)+3bc(b+c)+3ac(a+c)+6abc$

Mặt khác  $6abc\leq 2(a^3+b^3+c^3)$

                 $3ab\leq 3(a^2+b^2-ab)\Rightarrow 3ab(a+b)\leq 3(a^2+b^2-ab)(a+b)=3(a^3+b^3)$

Tương tự mấy cái khác rồi cộng vế theo vế

Suy ra $P^3\leq 9(a^3+b^3+c^3)$ (đpcm)

Bài 72 

ở bài 71 và 72 ta đã có $P^2\geq 12\sqrt{3}S$

                              $a^3+b^3+c^3\geq \frac{P^3}{9}$$a^3+b^3+c^3\geq \frac{P^3}{9}$

Suy ra     $a^3+b^3+c^3\geq \frac{P}{9}.12\sqrt{3}S=\frac{4\sqrt{3}}{3}SP$   (đpcm)  :D  :D  :D


:ohmy: [Không tồn tại các nghiệm nguyên khác không x, y, và z thoả mãn xn + yn = zn trong đó n là một số nguyên lớn hơn 2.  (FERMAT)  :ohmy: 

 

 

 

 






Được gắn nhãn với một hoặc nhiều trong số những từ khóa sau: bất đẳng thức, holder, cosi, bunhiacopxki

1 người đang xem chủ đề

0 thành viên, 1 khách, 0 thành viên ẩn danh